1984 미국수학올림피아드 3번문제

주어진 예각 $\theta$에 대해, 3차원 공간의 다섯 점 $P$, $A$, $B$, $C$, $D$는 $\angle APB = \angle BPC = \angle CPD = \angle DPA = \theta$ 를 만족한다. $\angle APC + \angle BPD$ 의 최대값과 최소값을 구하여라.

GD Star Rating
loading...